LSAT and Law School Admissions Forum

Get expert LSAT preparation and law school admissions advice from PowerScore Test Preparation.

 Administrator
PowerScore Staff
  • PowerScore Staff
  • Posts: 8916
  • Joined: Feb 02, 2011
|
#61060
Please post your questions below!
 lanereuden
  • Posts: 147
  • Joined: May 30, 2019
|
#65223
Why is A wrong? I thought if it costs less, then that would cement the idea for creating a tunnel that is modest, i.e. cheap but sufficiently built. Is it solely because of the language "design" that makes it wrong? That is, would A be correct if it said "build" rather than "design"? Also, someone noted to me that the budget aspect here is not fully expressed, and it seems to me that you are either at, over, or under-budget, so when they say they cannot put over the budget, that means at or under-budget. So I suppose, is this another aspect to why A is wrong--i.e. to be correct, it should have said cost as much as or less than the amount budgeted?
 James Finch
PowerScore Staff
  • PowerScore Staff
  • Posts: 943
  • Joined: Sep 06, 2017
|
#65227
Hi Lane,

There are three contending factors given in the stimulus: budget, "majesty" and convenience to commuters. The budget can't be increased, and the current design is majestic, but lacks a feature that is convenient to commuters. Without an increase in the budget, we can't have both a majestic station and a convenient one, so the stimulus is arguing that the design should be made less majestic in order to accommodate the feature that will make it more convenient. We're then tasked with strengthening this argument using a principle, which I prephrased as "Commuter convenience is a greater priority than majesty;" note the comparison element, which is crucial for allowing the argument to effectively choose one over the other in a situation where we can't have both.

(A) is effectively a restatement of part of the conclusion ("building a more modest station") rather than an actual reason/premise to do so. Again, the logical gap in the stimulus that we need to fill is the prioritization of commuter convenience over station majesty. (A) doesn't do this, while (C) does, making (C) the correct answer choice.

Hope this clears things up!
 snowy
  • Posts: 73
  • Joined: Mar 23, 2019
|
#65311
Is B wrong because “should consider” is too weak compared to the conclusion of the stimulus? I got this one correct but just wanted to make sure that I ruled out B for the right reason since it was my other contender.

Thank you!
User avatar
 Dave Killoran
PowerScore Staff
  • PowerScore Staff
  • Posts: 5853
  • Joined: Mar 25, 2011
|
#65315
Hi Snowy,

If everything else in this answer was perfect, I could probably live with "should consider" although I would prefer it be stronger. However, even though I don't love that part of the answer, it's the remainder—or perhaps better said, what is missing—that is the issue here. This answer focuses only on the idea that additions that make the station more convenient should be considered, but the stimulus was clearly focused on how considering those the addition should then spur a more modest redesign. This answer doesn't address the redesign aspect at all, and that is the part that most troubles me since we are asked to most justify the reasoning in the argument, which concludes that a more modest station should be built. Compare this answer to (C), and note how (C) successfully address both the convenience and redesign issues.

Please let me know if that helps. Thanks!
 snowy
  • Posts: 73
  • Joined: Mar 23, 2019
|
#65394
Got it, thank you for confirming Dave!

Get the most out of your LSAT Prep Plus subscription.

Analyze and track your performance with our Testing and Analytics Package.